Math, asked by Anonymous, 15 days ago

Prove the following
\sf \dfrac{tan \theta}{1- cot \theta} + \dfrac{cot \theta}{1 -tan \theta} = 1 + \tan \theta + \cot \theta

Answers

Answered by kailashmannem
65

 \Large{\bf{\orange{\mathfrak{\dag{\underline{\underline{To \: Prove:-}}}}}}}

\sf \dfrac{tan ∅}{1- cot ∅} + \dfrac{cot ∅}{1 -tan ∅} = 1 + tan ∅ + cot ∅

 \Large{\bf{\red{\mathfrak{\dag{\underline{\underline{Proof:-}}}}}}}

  • Taking LHS,

\sf \dfrac{tan ∅}{1- cot ∅} + \dfrac{cot ∅}{1 -tan ∅}

  • This implies that,

 \sf \dfrac{tan ∅}{1 \: - \: cot ∅} \: - \: \Big(\dfrac{cot ∅}{tan ∅ \: + \: 1}\Big)

 \sf \dfrac{tan ∅}{1 \: - \: cot ∅} \: - \: \dfrac{cot ∅}{tan ∅ \: - \: 1}

  • We know that,

 \sf cot ∅ \: = \: \dfrac{1}{tan ∅}

  • Substituting the values,

 \sf \dfrac{tan ∅}{1 \: - \: \dfrac{1}{tan ∅}} \: - \: \dfrac{\dfrac{1}{tan ∅}}{tan ∅ \: - \: 1}

 \sf \dfrac{tan ∅}{\dfrac{tan ∅ \: - \: 1}{tan ∅}} \: - \: \dfrac{1}{tan ∅ \: (tan ∅ \: - \: 1)}

 \sf \dfrac{tan^2 ∅}{tan ∅ \: - \: 1} \: - \: \dfrac{1}{tan ∅ \: (tan ∅ \: - \: 1)}

  • Taking  \sf \dfrac{1}{tan ∅ \: - \: 1} out,

 \sf \dfrac{1}{tan ∅ \: - \: 1} \: \Big(\dfrac{tan^2 ∅}{1} \: - \: \dfrac{1}{tan ∅}\Big)

  • Taking LCM,

 \sf \dfrac{1}{tan ∅ \: - \: 1} \: \Big(\dfrac{tan^3 ∅ \: - \: 1}{tan ∅}\Big)

  • We know that,

  • a³ - b³ = (a - b)(a² + ab + b²)

 \sf \dfrac{1}{tan ∅ \: - \: 1} \: \bigg(\dfrac{(tan ∅ \: - \: 1)(tan^2 ∅ \: + \: tan ∅ \: + \: 1^2)}{tan ∅}\bigg)

 \sf \dfrac{1}{\cancel{tan ∅ \: - \: 1}} \: \bigg(\dfrac{\cancel{(tan ∅ \: - \: 1)}(tan ^2 ∅ \: + \: tan ∅ \: + \: 1^2)}{tan ∅}\bigg)

 \sf 1 \: \bigg(\dfrac{tan^2 ∅ \: + \: tan ∅ \: + \: 1^2}{tan ∅}\bigg)

 \sf \dfrac{tan^2 ∅ \: + \: tan ∅ \: + \: 1}{tan ∅}

 \sf \dfrac{tan^2 ∅}{tan ∅} \: + \: \dfrac{tan ∅}{tan ∅} \: + \: \dfrac{1}{tan ∅}

 \sf \dfrac{tan^{\cancel{2}} ∅}{\cancel{tan ∅}} \: + \: \dfrac{\cancel{tan ∅}}{\cancel{tan ∅}} \: + \: \dfrac{1}{tan ∅}

 \sf tan ∅ \: + \: 1 \: + \: cot ∅

  • LHS = RHS

  • Hence, proved.

 \Large{\bf{\purple{\mathfrak{\dag{\underline{\underline{Important \: events \: of \: the \: answer:-}}}}}}}

  •  \sf Making \: the \: equation \: \dfrac{tan ∅}{1 \: - \: cot ∅} \: - \: \dfrac{cot ∅}{tan ∅ \: - \: 1}

  • Substituting  \sf \dfrac{1}{tan ∅} in place of cot ∅

  • Taking out  \sf \dfrac{1}{tan ∅ \: - \: 1} common.

  • Using a³ - b³ = (a² + ab + b²)(a - b)

  • Cancelling tan ∅ - 1
Answered by BrainlyRish
69

Given : \sf \dfrac{tan \theta}{1- cot \theta} + \dfrac{cot \theta}{1 -tan \theta} = 1 + \tan \theta + \cot \theta

Exigency To Prove :  \sf \dfrac{tan \theta}{1- cot \theta} + \dfrac{cot \theta}{1 -tan \theta} = 1 + \tan \theta + \cot \theta [ L.H.S = R.H.S ]

⠀⠀⠀⠀⠀━━━━━━━━━━━━━━━━━━━⠀

\qquad \dag\:\:\bigg\lgroup \sf{ \sf \dfrac{tan \theta}{1- cot \theta} + \dfrac{cot \theta}{1 -tan \theta} = 1 + \tan \theta + \cot \theta  }\bigg\rgroup \\\\

Here ,

  • \bf L.H.S = \:\sf \dfrac{tan \theta}{1- cot \theta} + \dfrac{cot \theta}{1 -tan \theta}
  • \bf R.H.S = \sf  1 + \tan \theta + \cot \theta

⠀⠀⠀⠀⠀⠀\underline {\boldsymbol{\star\:Now \: By \: Solving \: the \: L.H.S \:  \::}}\\

\qquad  :\implies \bf L.H.S = \:\sf \dfrac{tan \theta}{1- cot \theta} + \dfrac{cot \theta}{1 -tan \theta}

\qquad  :\implies  \sf \dfrac{tan \theta}{1- cot \theta} + \dfrac{cot \theta}{1 -tan \theta}

\qquad  :\implies  \sf \dfrac{tan \theta}{1- cot \theta} - \dfrac{cot \theta}{tan \theta-1}

\dag\:\:\it{ As,\:We\:know\:that\::}\\

  •  \sf cot \theta  = \dfrac {1}{cot \theta} \\

⠀⠀⠀⠀⠀⠀\underline {\boldsymbol{\star\:Now \: By \: Substituting \: this \::}}\\

\qquad  :\implies  \sf \dfrac{tan \theta}{1- cot \theta} - \dfrac{cot \theta}{tan \theta-1}

\qquad  :\implies  \sf \dfrac{tan \theta}{1- \dfrac{1}{tan \theta}} - \dfrac{\dfrac{1}{tan \theta }}{tan \theta-1}

\qquad  :\implies  \sf \dfrac{tan \theta}{ \dfrac{tan \theta - 1}{tan \theta}} - \dfrac{\dfrac{1}{tan \theta }}{tan \theta-1}

\qquad  :\implies  \sf \dfrac{tan \theta}{ \dfrac{tan \theta - 1}{tan \theta}} - \dfrac{1}{tan \theta (tan \theta-1)}

\qquad  :\implies  \sf \dfrac{tan^2 \theta}{ tan \theta - 1} - \dfrac{1}{tan \theta (tan \theta-1)}

  • Taking  \bf \dfrac {1}{tan \theta -1 } as common :

\qquad  :\implies  \sf \dfrac{tan^2 \theta}{ tan \theta - 1} - \dfrac{1}{tan \theta (tan \theta-1)}

\qquad  :\implies  \sf \dfrac{1}{tan \theta - 1} \bigg[  tan^2 \theta - \dfrac{1}{tan \theta } \bigg]

\qquad  :\implies  \sf \dfrac{1}{tan \theta - 1} \bigg[  \dfrac{tan^3 \theta - 1 }{tan \theta } \bigg]

\dag\:\:\it{ As,\:We\:know\:that\::}\\

  • Algebraic Indentity : a³ - b³ = (a² + ab + b² ) (a - b)

⠀⠀⠀⠀⠀⠀\underline {\boldsymbol{\star\:Now \: By \: Applying \: this \:Indentity \::}}\\

\qquad  :\implies  \sf \dfrac{1}{tan \theta - 1} \bigg[  \dfrac{tan^3 \theta - 1 }{tan \theta } \bigg]

\qquad  :\implies  \sf \dfrac{1}{tan \theta - 1} \bigg[  \dfrac{(tan^2 \theta + tan \theta + 1^2 ) ( tan \theta - 1 ) }{tan \theta } \bigg]

\qquad  :\implies  \sf \dfrac{1}{tan \theta - 1}  \times  \dfrac {(tan^2 \theta + tan \theta + 1^2 ) ( tan \theta - 1 ) }{tan \theta }

\qquad  :\implies  \sf \dfrac{1}{\cancel {tan \theta - 1}}  \times  \dfrac {(tan^2 \theta + tan \theta + 1^2 )\cancel {( tan \theta - 1 )} }{tan \theta }

\qquad  :\implies  \sf   \ \dfrac {tan^2 \theta + tan \theta + 1^2 }{tan \theta }

\qquad  :\implies  \sf   \ \dfrac {tan^2 \theta + tan \theta + 1 }{tan \theta }

\qquad  :\implies  \sf   \ \dfrac {tan^2 \theta}{tan \theta } + \cancel {\dfrac{ tan \theta}{tan \theta }}+\dfrac{  1 }{tan \theta }

\qquad  :\implies  \sf   \ \cancel {\dfrac {tan^2 \theta}{tan \theta }} +1 + \dfrac{  1 }{tan \theta }

\qquad  :\implies  \sf   \  tan \theta  +1 + \dfrac{  1 }{tan \theta }

\qquad  :\implies  \sf   \  tan \theta  +1 + cot \theta

\qquad  :\implies    \bf L.H.S \:  :  \sf 1 + tan \theta  + cot \theta

Therefore,

  • \qquad  :\implies    \bf L.H.S \:=    \sf 1 + tan \theta  + cot \theta

  • \qquad  :\implies    \bf R.H.S \: =   \sf 1 + tan \theta  + cot \theta

⠀⠀⠀⠀⠀━━ As , We can see that here L.H.S = R.HS .

⠀⠀⠀⠀⠀\therefore {\underline {\bf{ Hence, \:Proved\:}}}\\\\

⠀⠀⠀⠀⠀━━━━━━━━━━━━━━━━━━━⠀

Similar questions